LSAT and Law School Admissions Forum

Get expert LSAT preparation and law school admissions advice from PowerScore Test Preparation.

 Administrator
PowerScore Staff
  • PowerScore Staff
  • Posts: 8919
  • Joined: Feb 02, 2011
|
#32472
Complete Question Explanation

Assumption—SN. The correct answer choice is (E)

Here, the author starts the stimulus with the “some people say...” technique, telling us that, periodically, the public complains about predatory pricing, which is defined as being when “a company deliberately sells its products at prices low enough to drive its competitors out of business.” Despite this periodic outcry, the author concludes, in the second sentence, that predatory pricing (i.e., “this practice”) should be acceptable.

The author’s sole reason for reaching this conclusion is that although the practice drives the company’s competitors out of business, the continuing threat of new competition keeps the company from “raising its prices to unreasonable levels.” Essentially, by having just one premise leading to its definitive conclusion, the author is acting as if this fact, by itself, is enough to show the conclusion is valid. In other words, the author treats the premise as if it is a sufficient condition that shows the conclusion must be the case. We could diagram this relationship as:


PPC =practice prevents a company from raising prices to unreasonable levels
PA = practice should be acceptable

..... ..... ..... ..... Sufficient ..... ..... Necessary

..... ..... ..... ..... PPC ..... :arrow: ..... PA


The question stem tells us that this is an Assumption question. Our prephrase is that the correct answer choice will supply this conditional rule the author implicitly applied in reaching the conclusion.

Answer choice (A): It is not required that competitors actually enter the market. The threat of renewed competition is what keeps a company from raising prices unreasonably.

Answer choice (B): By definition, it appears that several competing companies cannot simultaneously engage in predatory pricing, not for long at least. However, even if they were to do so, the conclusion was concerned about preventing companies from raising prices to unreasonable levels. If multiple companies engaged in predatory pricing, the result would be to lower prices to unreasonable levels.

Answer choice (C): The size of the companies is not relevant to the argument.

Answer choice (D): While it was the threat of competition that the author focused on here, the conclusion does not require that the threat of competition be the only thing that keeps companies from raising prices.

Answer choice (E): This is the correct answer choice, because it provides a rule that, while not precisely the same as the rule applied by the author, is very similar and has the same effect. We can diagram this answer choice as:


PPNRUP = pricing practice does not result in unreasonable prices
PA = practice should be acceptable

..... ..... ..... ..... Sufficient ..... ..... Necessary

..... ..... ..... ..... PPNRUP ..... :arrow: ..... PA
 mpoulson
  • Posts: 148
  • Joined: Mar 25, 2016
|
#25430
Hello,

I wanted help for question 22. What specifically makes the answer E? Was it that the other items help the argument but are essential to making it valid? Please provide a simple detailed explanation. Thank you and appreciate the help.

- Micah
 Nikki Siclunov
PowerScore Staff
  • PowerScore Staff
  • Posts: 1362
  • Joined: Aug 02, 2011
|
#25440
Micah,

The author begins by presenting an argument that will ultimately be rejected. The author argues that predatory pricing should be acceptable. Why? Because although the practice drives the company’s competitors out of business, the continuing threat of new competition keeps the prices at reasonable levels.

This may be so, but who is to say that keeping prices at reasonable levels is enough of a justification for accepting the practice of predatory pricing? The author considers the benefits, but not the costs, of the proposed pricing strategy.

When in doubt, apply the Assumption Negation Technique to your contender and analyze the implication of the logical opposite on the conclusion. What if not every pricing practice that keeps prices at a reasonable level should be accepted? In other words, what if keeping prices at a reasonable level were not enough to justify accepting a pricing strategy? This would immediately weaken the conclusion of the argument, proving that answer choice (E) states an assumption upon which the argument depends.

Hope this helps!

Thanks,
 asuper
  • Posts: 10
  • Joined: Jul 21, 2018
|
#48205
Could you explain, in a different manner, why answer D is incorrect? I guess I was drawn to the answer because it specified "the threat of competition" which is specified in the conclusion. Is another way to eliminate this answer to consider that it also specifies "competition" as well which is not stated in the conclusion? -Thank you
 Jennifer Janowsky
PowerScore Staff
  • PowerScore Staff
  • Posts: 90
  • Joined: Aug 20, 2017
|
#48664
asuper wrote:Could you explain, in a different manner, why answer D is incorrect? I guess I was drawn to the answer because it specified "the threat of competition" which is specified in the conclusion. Is another way to eliminate this answer to consider that it also specifies "competition" as well which is not stated in the conclusion? -Thank you
Good question, ASuper!

"(D) It is only competition or the threat of competition that keeps companies from raising prices."

To put simply, (D) is not a terrible answer, and might even have been the correct one if (E) didn't exist.

However, the author's focus is not on prices staying low, but on "acceptability" of the practice: "This practice clearly should be acceptable." Although low prices are given as evidence by the author, prices staying low, and the causes of them staying low, were not the main point he was trying to make.

What does he give as the only evidence that the practice is still acceptable? That the prices will stay low. However, prices being low still need to be connected back to the main point, and (E) does that in saying practices are acceptable so long as prices stay reasonable.

Doesn't result in unreasonable prices = Acceptable practice, it was the last link the argument needed to make sense.

I hope that makes sense, thanks for your question!
 jayzbrisk
  • Posts: 12
  • Joined: Mar 19, 2019
|
#65056
If I am understanding choice E correctly, then it is not something that the argument depends on being true, (if you apply the negation test it will not refute the authors argument) which may be a good answer for an assumption question that asks which would allow the conclusion to be properly drawn. But doesn't this question ask for a necessary assumption which would need to pass the negation test?
 James Finch
PowerScore Staff
  • PowerScore Staff
  • Posts: 943
  • Joined: Sep 06, 2017
|
#65401
Hi Jay,

(E) negated will absolutely lead to the negation of the conclusion; in fact, it is the only answer choice that will. A big clue here is that the conclusion introduces a new element, acceptability, which isn't actually a part of the premises (although hinted at by the background info in the first sentence). That makes this a Supporter Assumption, where the correct answer choice will link the premise (about threat of competition keeping prices reasonable) to acceptability.

Only one answer choice does this, so we only need to test it (E):

Pricing practices that don't result in unreasonable prices should not be acceptable :arrow: Predatory Pricing should not be acceptable

Combine the negated assumption with the premise given and we see that it leads conclusively to the negation of the conclusion, meaning this is a necessary conclusion to the argument and thus the correct answer choice.

Hope this clears things up!
 jayzbrisk
  • Posts: 12
  • Joined: Mar 19, 2019
|
#65419
Can you help me clarify why the negation is ;

Any pricing practice that does not result in unreasonable prices should not be acceptable.

As opposed to ;

Any pricing practice that does result in unreasonable prices should be acceptable.
 Jay Donnell
PowerScore Staff
  • PowerScore Staff
  • Posts: 144
  • Joined: Jan 09, 2019
|
#65420
Hi Jayzbrisk!

Deciphering the proper way to negate answer choices can often be frustrating, but the value it offers in allowing us to ensure we select the correct response in an Assumption question makes it a battle worth fighting!

A rule of thumb of sorts that I use in my classes is to try and understand the break in between where the answer choice language selects a term or concept and where it begins saying something about that term or concept.

For example:

Restaurants that allow you to bring your own wine are great options for cheap dates.

In this example, that 'break' I mentioned earlier would be right at the word "are." That 'break' in the sentence is also typically the most assertive point of the statement, and therefore exactly what should be attacked in efforts to negate the answer choice.

The negation above would then become:

Restaurants that allow you to bring your own wine are NOT great options for cheap dates.


If it's unclear where in the sentence you can divide up the claim to find where to negate, you can often find clues in the use of quantified indicators (all, some, most, none, etc).

For example:

No made-for-television movies on the Lifetime network are worth watching.

In this case, the 'No' is the most assertive portion of the claim, and therefore should be negated to its logical opposite of 'some.'

Negation:

SOME made-for-television movies on the Lifetime network are worth watching.


And here's the big kicker. If you are unable to decide which declarative portion of the answer should be attacked to negate (often adding or removing a 'not' in the central position of the sentence) or have either no quantified indicators to break (or sometimes too many to be able to decide which to negate), fear not! We always have the ability of using our backup parachute, the blanket negation phrase of: "It's not true that..."

By thinking (or saying aloud, when possible in practice) "It's not true that..." before reading the answer choice exactly as it is, you will convince your brain that this is no longer a fact that we can trust or accept, which in essence is exactly what happens when we negate an answer choice.

In use for the examples above:

"It's not true that... restaurants that allow you to bring your own wine are great options for cheap dates.

"It's not true that... no made-for-television movies on the Lifetime network are worth watching.

Can you see how effective this can be in utilizing the Assumption Negation Technique?

And, perhaps most importantly, let's use this backup parachute for the answer choice in question:

It's not true that... any pricing practice that does not result in unreasonable prices should be acceptable.

Doesn't that sound a lot more like the proper negation discussed earlier in this thread?

Any pricing practice that does not result in unreasonable prices should not be acceptable.

As opposed to:

Any pricing practice that does result in unreasonable prices should be acceptable.



Hopefully this helps to clear up the tools available to you in either searching for that power point in the sentence between selection and declaration, using quantified indicators as the source of the negation attack, or perhaps most usefully, the all-inclusive savior, blanket negation phrase: "It's not true that..."

Hope that helps!
 t_m6289
  • Posts: 7
  • Joined: Nov 06, 2019
|
#71762
I am curious as to why the correct answer choice isn't too strong for it to be a necessary assumption. To me, it seems that "any" is too strong to necessarily be true when the conclusion is true; the necessary assumption would be "Some predatory pricing practices that do not result in unreasonable prices are acceptable." Maybe some are unacceptable, that seems like it would still be fine for the author's argument.

What's wrong with my thinking here?

Get the most out of your LSAT Prep Plus subscription.

Analyze and track your performance with our Testing and Analytics Package.